Practice Questions for the LSAT Logical Reasoning

lsat logical reasoning practice
Getty Images | Tanya Constantine

The questions in this section are based on the reasoning contained in brief statements or passages. For some questions, more than one of the choices could conceivably answer the question. However, you are to choose the best answer; that is, the response that most accurately and completely answers the question. You should not make assumptions that are by commonsense standards implausible, superfluous, or incompatible with the passage. After you have chosen the best answer, blacken the corresponding space on your answer sheet.

Question 1

Biologists attached a radio transmitter to one of a number of wolves that had been released earlier in the White River Wilderness Area as part of a relocation project. The biologists hoped to use this wolf to track the movements of the whole pack. Wolves usually range over a wide area in search of prey, and frequently follow the migrations of their prey animals. The biologists were surprised to find that this particular wolf never moved more than five miles away from the location in which it was first tagged.

Which one of the following, if true, would by itself most help to explain the behavior of the wolf tagged by the biologists?

A. The area in which the wolves were released was rocky and mountainous, in contrast to the flat, heavily-wooded area from which they were taken. 

B. The wolf had been tagged and released by the biologists only three miles away from a sheep ranch that provided a large, stable population of prey animals.

C. The White River Wilderness Area had supported a population of wolves in past years, but they had been hunted to extinction.

D. Although the wolves in the White River Wilderness Area were under government protection, their numbers had been sharply reduced, within a few years of their release, by illegal hunting.

E. The wolf captured and tagged by the biologists had split off from the main pack whose movements the biologists had hoped to study, and its movements did not represent those of the main pack.

Answer below. Scroll down.

Question 2

As any economist knows, healthy people pose less of an economic burden to society than unhealthy people. Not surprisingly, then, every dollar our state government spends on prenatal care for undocumented immigrants will save taxpayers of this state three dollars.

Which of the following, if true, would best explain why the statistics cited above are not surprising?

A. The state’s taxpayers pay for prenatal care of all immigrants.

B. Babies born in this state to undocumented immigrant parents are entitled to infant care benefits from the state.

C. State benefits for prenatal care serve to promote undocumented immigration.

D. Babies whose mothers did not receive prenatal care.are just as healthy as other babies.

E. Pregnant women who do not receive prenatal care are more likely to experience health problems than other pregnant women.

Question 3

Beautiful beaches attract people, no doubt about it. Just look at this city’s beautiful beaches, which are among the most overcrowded beaches in Florida.

Which of the following exhibits a pattern of reasoning most similar to the one exhibited in the argument above?

A. Moose and bear usually appear at the same drinking hole at the same time of day. Therefore, moose and bear must grow thirsty at about the same time.

B. Children who are scolded severely tend to misbehave.more often than other children. Hence if a child is not scolded severely that child is less likely to misbehave.

C. This software program helps increase the work efficiency of its users. As a result, these users have more free time for other activities.

D. During warm weather, my dog suffers from fleas more so than during cooler weather. Therefore, fleas must thrive in a warm environment.

E. Pesticides are known to cause anemia in some people. However, most anemic people live in regions where pesticides are not commonly used.

Answers to LSAT Logical Reasoning Questions

Question 1:

Most wolves range over a wide area in search of prey; this particular wolf hung around the same area. An explanation that immediately suggests itself is that this particular wolf found enough prey in this area, so it didn’t have to run all over looking for food. This is the tack taken by B. If the wolf had a large stable population of sheep on which to pray in the immediate vicinity, there was no need for it to range over a wide territory looking for food.

A doesn’t have much direct bearing on this particular wolf’s lack of mobility. While it’s true that a wolf might find it harder to move around in the mountainous country, the stimulus says that wolves, in general, tend to cover great distances in search of food. There’s no hint that a wolf in a mountainous area should prove an exception to this rule.

C is irrelevant: While the White River Wilderness Area may once have supported a population of wolves, knowing this does nothing to explain the behavior of this particular wolf.

D, if anything, gives what seems to be a reason for our wolf to make tracks and migrate somewhere else. Certainly, D doesn’t explain why our wolf didn’t follow usual wolf hunting methods.

E answers the wrong question; it would help explain why the naturalists couldn’t use our wolf to study the movements of the larger pack. However, we haven’t been asked that; we want to know why this specific wolf didn’t behave the way wolves usually do.

Question 2

The argument relies on the unstated assumption that prenatal care results in better health and therefore less cost to society. E helps to affirm this assumption.

A is irrelevant to the argument, which makes no distinction between undocumented immigrants and other immigrants.

B describes benefits that might decrease the overall tax burden, but only if the prenatal care program serves to reduce the amount of infant-care benefits paid. The argument does not inform us whether this is the case. Thus it is impossible to assess the extent to which B would explain how the prenatal care would save the taxpayers money.

C actually renders the statistics more surprising, by providing evidence that prenatal care will add to society’s economic burden.

D also renders the statistics more surprising, by providing evidence that the cost of the prenatal care program will not be offset by a particular health benefit—a benefit which would lessen the taxpayers\’ economic burden.

Question 3

The correct response to Question 3 is (D). The original argument bases a conclusion that one phenomenon causes another on an observed correlation between the two phenomena. The argument boils down to the following:

Premise: X (beautiful beach) is correlated with Y (crowd of people).
Conclusion: X (beautiful beach) causes Y (crowd of people).

Answer choice (D) demonstrates the same pattern of reasoning:

Premise: X (warm weather) is correlated with Y (fleas).
Conclusion: X (warm weather) causes Y (fleas).

(A) demonstrates a different pattern of reasoning than the original argument:

Premise: X (moose at the drinking hole) is correlated with Y (bears at the drinking hole).
Conclusion: X (moose) and Y (bear) are both caused by Z (thirst).

(B) demonstrates a different pattern of reasoning than the original argument:

Premise: X (scolding children) is correlated with Y (misbehavior among children).
Assumption: Either X causes Y, or Y causes X.
Conclusion: Not X (no scolding) will be correlated with not Y (no misbehavior).

(C) demonstrates a different pattern of reasoning than the original argument:

Premise: X (software program) causes Y (efficiency).
Assumption: Y (efficiency) causes Z (free time).
Conclusion: X (software program) causes Z (free time).

(E) demonstrates a different pattern of reasoning than the original argument. In fact, (E) is not a complete argument; it contains two premises but no conclusion:

Premise: X (pesticides) causes Y (anemia).
Premise: Not X (pesticide-free regions) is correlated with Y (anemia). 

Format
mla apa chicago
Your Citation
Roell, Kelly. "Practice Questions for the LSAT Logical Reasoning." ThoughtCo, Oct. 6, 2021, thoughtco.com/lsat-logical-reasoning-practice-questions-4075528. Roell, Kelly. (2021, October 6). Practice Questions for the LSAT Logical Reasoning. Retrieved from https://www.thoughtco.com/lsat-logical-reasoning-practice-questions-4075528 Roell, Kelly. "Practice Questions for the LSAT Logical Reasoning." ThoughtCo. https://www.thoughtco.com/lsat-logical-reasoning-practice-questions-4075528 (accessed March 28, 2024).